Difference between revisions of "2021 Fall AMC 10A Problems/Problem 21"

m (Solution 1 (Multinomial Numbers))
(redirect to 12A #18)
(Tag: New redirect)
 
(2 intermediate revisions by 2 users not shown)
Line 1: Line 1:
==Problem==
+
#REDIRECT [[2021_Fall_AMC_12A_Problems/Problem_18]]
Each of the <math>20</math> balls is tossed independently and at random into one of the <math>5</math> bins. Let <math>p</math> be the probability that some bin ends up with <math>3</math> balls, another with <math>5</math> balls, and the other three with <math>4</math> balls each. Let <math>q</math> be the probability that every bin ends up with <math>4</math> balls. What is <math>\frac{p}{q}</math>?
 
 
 
<math>\textbf{(A)}\ 1 \qquad\textbf{(B)}\  4 \qquad\textbf{(C)}\  8 \qquad\textbf{(D)}\
 
12 \qquad\textbf{(E)}\ 16</math>
 
 
 
==Solution 1 (Multinomial Coefficients)==
 
For simplicity purposes, we assume that the balls are indistinguishable and the bins are distinguishable.
 
 
 
Let <math>d</math> be the number of ways to distribute <math>20</math> balls into <math>5</math> bins. We have
 
<cmath>p=\frac{5\cdot4\cdot\binom{20}{3,5,4,4,4}}{d} \text{ and } q=\frac{\binom{20}{4,4,4,4,4}}{d}.</cmath> Therefore, the answer is <cmath>\frac pq=\frac{5\cdot4\cdot\binom{20}{3,5,4,4,4}}{\binom{20}{4,4,4,4,4}}=\frac{5\cdot4\cdot\frac{20!}{3!5!4!4!4!}}{\frac{20!}{4!4!4!4!4!}}=\frac{5\cdot4\cdot(4!4!4!4!4!)}{3!5!4!4!4!}=\frac{5\cdot4\cdot4}{5}=\boxed{\textbf{(E)}\ 16}.</cmath>
 
 
 
<u><b>Remark</b></u>
 
 
 
By the stars and bars argument, we get <math>d=\binom{20+5-1}{5-1}=\binom{24}{4}.</math>
 
 
 
~MRENTHUSIASM
 
 
 
==Solution 2 (Simple) ==
 
Since both of the boxes will have <math>3</math> boxes with <math>4</math> balls in them, we can leave those out. There are <math>\binom {6}{3}</math> = <math>20</math> ways to choose where to place the <math>3</math> and the <math>5</math>. After that, there are <math>\binom {8}{3} = 56</math> ways to put the <math>3</math> and <math>5</math> balls being put into the boxes. For the <math>4,4,4,4,4</math> case, after we canceled the <math>4,4,4</math> out, we have <math>\binom {8}{4}</math> = <math>70</math> ways to put the <math>4</math> balls inside the boxes. Therefore, we have <math>\frac {56\cdot 20}{70}</math> which is equal to <math>8 \cdot 2</math> = <math>\boxed {(E)16}</math>
 
 
 
~Arcticturn
 
 
 
==Video Solution by Punxsutawney Phil==
 
https://YouTube.com/watch?v=bvd2VjMxiZ4
 
 
 
==See Also==
 
{{AMC10 box|year=2021 Fall|ab=A|num-b=20|num-a=22}}
 
{{MAA Notice}}
 

Latest revision as of 20:50, 23 November 2021